The difference of a number and 6is the same as 5 times

Answers

Answer 1

Answer:

let difference be x

Step-by-step explanation:

you can solve for x by keeping the 2 statesments as conditions.

then you solve for both equations together by adding subtracting or multiplying.

eg: let x be the number.

the. proceed with conditions as equations

then solve

then your answer is here


Related Questions

Which formulas could you use to find the volume of a right circular
cone?
OA) V = B-h where B = I-w
OB) V=B-h where B=1r'
OC) V=B-h where B = Tr
OD) V=B-h where B=1-w

Answers

Answer:

The formula for the volume of a cone is V=1/3hπr².

A water sample shows 0.033 grams of some trace element for every cubic centimeter of water. Marques uses a container in the shape of a right cylinder with a diameter of 13.2 cm and a height of 19.7 cm to collect a second sample, filling the container all the way. Assuming the sample contains the same proportion of the trace element, approximately how much trace element has Marques collected? Round your answer to the nearest tenth.

Answers

Answer:

89.0 grams of trace element

Step-by-step explanation:

Step 1

Find the volume of the sample container.

Given:

Container is a right cylinder with d = 13.2 cm and h = 19.7 cm

Volume formula:

V = πr²h = πd²h/4

Substitute to get:

V = π(13.2)²(19.7)/4 = 2695.9 cm³ (rounded)

Step 2

Find the volume of the trace element, using the given proportion:

2695.9*0.033 = 89.0 grams (rounded)

The least common denominator of 5/6, 2/3, and 3/5 is:

Answers

Answer:

30

Step-by-step explanation:

¿Cuáles son las soluciones x1 y x2 de la ecuación x2 − 5x − 6 = 0?

Answers

Answer:

x1 = 4 y x2= 2

Step-by-step explanation:

escuchame

x2 - 5x -6 = 0

(4)^2 - 5(2) - 6 = 0

16 - 10 - 6 = 0

6 - 6 = 0

de nade :)

What is the value of x?



Enter your answer in the box.

...°

Answers

X=111

A 7 sided shape all the interior angles should add to 900 so if you add all the angles given you get 789 then subtract that from 900

Solve the problem 4(a+3)=-32

Answers

[tex]4(a + 3) = - 32 \\ 4a + 12 = - 32 \\ 4a = - 32 - 12 \\ 4a = - 44 \\ a = - 11[/tex]

Hope it helps

Please give brainliest

Step-by-step explanation:

4(a+3)=-32

4(a+3)/4=-32/4

(a+3)=-8

a+3-3=-8-3

a=-11

Pls help. !!!!!!!!!!!!!!!!!! (Give steps if you can)

Answers

Answer:

[tex]144 cm^{2}[/tex]

Step-by-step explanation:

Step 1: Find the true length of each side

We know that the perimeter is 56cm and the ratio of AD:AB:DC:BC is 5:12:6:5. It is safe to assume that this ratio is the simplified version, which means that if we multiply each of these numbers by a scale factor of [tex]x[/tex], then we get the true length of each side, so we would get [tex]5x, 12x, 6x, 5x[/tex] for the sides. We also know that when we add these numbers, we get the perimeter, which is 56. So, we can create the equation [tex]5x + 12x + 6x + 5x = 56[/tex]. Then, solve the equation for x, which would be 2, and go back and multiply each number with 2 to get the following:

AD = 5*2 = 10cm, AB = 12*2 = 24cm, DC = 6*2 = 12cm, BC = 5*2 = 10cm  

Step 2: Calculate the Height

Now we know the lengths of each side, lets take a look at the formula for the area of a trapezium: [tex]\frac{base_{1} + base_2}{2}*height[/tex]. So, the next step would be to calculate the height. We can draw the height by drawing a line straight down from the vertex D and another straight down from the vertex C. These lines are both the height, and will be the exact same length. We can calculate that the height is 8 because AB is 24 cm, but the length from the height at vertex D to the height at vertex C is 12 cm, so that means that the base of each triangle is 12/2 = 6 cm. Then we can use Pythagorean theorem to figure out that the height is 8cm.

Step 3: Calculate the Area

Lastly, we can calculate the area. The formula is [tex]\frac{base_{1} + base_2}{2}*height[/tex]. In this case, the numbers would be [tex]\frac{12 + 24}{2} * 8 = 144cm^{2}[/tex]

I know this is a lot, so let me know if you have any questions!

Which of the following points is NOT visible if the viewing window of a graphing utility is set at:
Xmin : 6
Xmaz: 6
Ymin : —6
Ymaz : 6

Answers

The points D(-7, 5), E(-8, -3), and H(10, -5) will not be visible the answers are D, E, and H.

What is graphing utility?

A graphing calculator is a portable computer that can plot graphs, solve multiple equations at once, and carry out other variable-related activities.

We have points shown in the coordinate plane.

As the window of a graphing utility is set at:

X(min) : -6

X(max): 6

Y(min) : -6

Y(max) : 6

The points out of 6 will not be visible.

We can draw a square as shown in the attached picture.

The points D(-7, 5), E(-8, -3), and H(10, -5) will not be visible.

Thus, the points D(-7, 5), E(-8, -3), and H(10, -5) will not be visible the answers are D, E, and H.

Learn more about the graphing utility here:

https://brainly.com/question/1549068

#SPJ1

I need help answering these questions for geometry :)

1. What happens to the measure of the interior angles as the number of sides of the polygon increases? Explain your answer.
2.What happens to the measure of the exterior angles as the number of sides of the polygon increases? Explain your answer.
4. Explain what happens to the total sum of interior angles as the number of sides in the polygon changes.
5. Explain what happens to the total sum of exterior angles as the number of sides in the polygon changes.

Answers

The answers to the question have been written below

The changes to a polygon

1. If the number of sides of the polygon should increase then the measure of the interior angle would increase by 180.

2. A regular polygon has the sum of its external angles as 360. This is solved as 360/n = 0

Hence the value of the exterior would move towards 0 as its sides increases.

4. As the total sum of the of the polygon changes, it would tend towards infinity.

5. The total sum of the exterior angles as the number of sides changes would tends towards 360 degrees.

Read more on polygon here:

https://brainly.com/question/1592456

#SPJ1

Answer the following

Answers

The set A satisfying the given inequality is A = (-[tex]\infty[/tex], -10].

What are some properties of an inequality relation?

Following are some facts which are true for an inequality relation:

Equal numbers can be added or subtracted from both sides of an inequality without affecting the inequality sign.The Inequality sign is unchanged if both sides are multiplied or divided by a positive number, but when multiplied or divided by a negative number the inequality sign is reversed.

[tex]\frac{5x-2}{8} - \frac{3x-5}{10} &\ge& x+y\\\\\Rightarrow\;\; \frac{13}{40}x + \frac{1}{4}&\ge& x+y\\\\\Rightarrow\;\;\;\;\;\; -\frac{27}{40}x &\ge & y - \frac{1}{4}\\\\\Rightarrow\;\;\;\;\;\;\;\;\;\; -x &\ge & \frac{40}{27}\left( y-\frac{1}{4} \right).\hspace{1cm}(1)[/tex]

Since y ∈ B, -2 ≤ y ≤ 7. So,

[tex]\;\;\;\;\;\;\;\,-2 - \frac{1}{4}\; \le\; y - \frac{1}{4} \;\le\; 7 - \frac{1}{4}\\\\\Rightarrow\;\;\;\;\;\;\;\;\; -\frac{9}{4}\; \le\; y - \frac{1}{4} \;\le\; \frac{27}{4}\\\\\Rightarrow\;\;\; -\frac{9}{4}\cdot \frac{40}{27} \;\le\; \frac{40}{27} \left( y-\frac{1}{4} \right) \;\le \;\frac{27}{4}\cdot \frac{40}{27}\\\\\Rightarrow\;\;\;\;\;\;\;\; -\frac{10}{3} \;\le\; \frac{40}{27}\left( y - \frac{1}{4} \right)\; \le\; 10.[/tex]

The set {-x | inequality (1) holds ∀ y ∈ B} is [10, [tex]\infty[/tex]) i.e.

10 ≤ -x ≤ [tex]\infty[/tex].

Multiplying -1 throughout gives

-10 ≥ x ≥ -[tex]\infty[/tex].

x, thus, lies in the range A = (-[tex]\mathbf{\infty}[/tex], -10}.

Learn more about the inequality here.

https://brainly.com/question/17801003

Disclaimer: The question was incomplete. Please find the full content below.

Question

Find the set A such that for x ∈ A

[tex]\frac{5x - 2}{8} - \frac{3x - 5}{10} \ge x + y[/tex]

∀y ∈ B = {y ∈ R | -2 ≤ y ≤ 7}.

#SPJ4

If an employer reduces the working week from 40 hours to 35 hours, with no
loss of weedy pay, calculate the percentage increase in the hourly rate of pay.

Answers

Answer:

14.285714285714 %

Step-by-step explanation:

suppose the weekly payment is of x dollars

Then

Previous payment : $(x/40) per hour

New payment : $(x/35) per hour

Then

The percentage increase in the hourly rate of pay is :

[tex]=\frac{\frac{x}{35} -\frac{x}{40} }{\frac{x}{40} } \times100[/tex]

[tex]=\frac{\frac{5x}{1400} }{\frac{x}{40} } \times100[/tex]

[tex]=\frac{5x}{1400} \times \frac{40}{x} \times100[/tex]

[tex]=\frac{200}{1400} \times100[/tex]

[tex]=\frac{1}{7} \times100[/tex]

[tex]=14.285714285714[/tex]

Question 10 of 10
Which of the following are necessary when proving that the diagonals of a
rectangle are congruent? Check all that apply.
A. Corresponding parts of similar triangles are similar.
B. Corresponding parts of congruent triangles are congruent.
C. Opposite sides are congruent.
D. Opposite sides are perpendicular.

Answers

I think the option is D, not sure tho
Yes the answers is D ….

What are the solutions to the following system of equations?
y = x² + 7x-5
2x - y = -9

Answers

Answer:

Step-by-step explanation:

y = x² + 7x - 5 ............ (1)

2x - y = - 9 ................ (2)

(1) ------> (2)

2x - ( x² + 7x - 5 ) = - 9

- x² - 5x + 14 = 0 ........ (3)

- 1 × (3)

x² + 5x - 14 = 0

( x + 7 )( x - 2 ) = 0

[tex]x_{1}[/tex] = - 7 and [tex]x_{2}[/tex] = 2

2( - 7) - [tex]y_{1}[/tex] = - 9 ⇒ [tex]y_{1}[/tex] = - 5

2(2) - [tex]y_{2}[/tex] = - 9 ⇒ [tex]y_{2}[/tex] = 13

The solutions of system are

( - 7, - 5 ) and ( 2, 13 )

What is the equivalent exponential expression for the radical expression below? √3+7​

Answers

Answer:

09 l

Step-by-step explanation:

p0-9

The equivalent exponential expression for the given radical expression √(3 + 7) is √10.

To convert the given radical expression √(3 + 7) into its equivalent exponential expression. This process involves using exponentiation to represent the radical form.

The given radical expression is √(3 + 7). To find its equivalent exponential expression, we need to express the radical in terms of an exponent.

Recall that a radical expression of the form √(a) is equivalent to raising 'a' to the power of 1/2. In this case, 'a' is the quantity inside the radical, which is 3 + 7. So, we can write:

√(3 + 7) = (3 + 7)^(1/2).

Next, we simplify the expression inside the parentheses:

3 + 7 = 10.

Now, the expression becomes:

(10)^(1/2).

To further simplify, we raise 10 to the power of 1/2. Recall that raising a number to the power of 1/2 is the same as finding the square root of that number. So:

(10)^(1/2) = √10.

Therefore, the equivalent exponential expression for the given radical expression √(3 + 7) is √10.

To summarize, we transformed the radical expression √(3 + 7) into its equivalent exponential expression √10 by using the rule that √(a) is equal to a^(1/2). By raising the expression inside the radical to the power of 1/2, we simplified it and found the square root of 10 as the final result.

To know more about Exponential Expression here

https://brainly.com/question/26540624

#SPJ2

Sandhu lights a candle at 6 pm. It is 30 cm high. At 7pm, the candle is 27 cm high. At 8pm, it is 24cm high. a) How many cm does the candle burn down every hour? b) How high is the candle at 11 pm?

Answers

2 water allowed going into cm high into squares

The data set represents the number of minutes dan spent on his homework each night. 30, 35, 35, 45, 46, 46, 52, 55, 57 which box plot correctly represents the data?

Answers

The box plot that represents the data is the first option.

What box plot correctly represents the data?

A box plot is used to study the distribution and level of a set of scores. The box plot consists of two lines and a box. The end of the first whisker represents the minimum number and the end of the second whisker represents the maximum number.

On the box, the first line to the left represents the lower (first) quartile. The next line on the box represents the median. The third line on the box represents the upper (third) quartile.

Minimum value = 30

Maximum value = 57

1st quartile = 1/4 (n + 1)

1/4 x 10 = 2.5th term = 35

Third quartile = 3/4 x (n + 1 )

3/4 x 10 = 7.5 term = 53

Median = 46

Please find attached the complete question. To learn more about box plots, please check: https://brainly.com/question/27215146

#SPJ1

PLEASE HURRY WILL GIVE POINTS AND BRAINLYEST TO FIRST RIGHT!!!

Answers

Answer:

(0, 4) and (-1, 0)

Step-by-step explanation:

Given system of equations:

[tex]\begin{cases}y=2x^2+6x+4\\y=-4x^2+4\end{cases}[/tex]

Solve by substitution

Substitute the first equation into the second:

[tex]\implies 2x^2+6x+4=-4x^2+4[/tex]

Add 4x² to both sides:

[tex]\implies 2x^2+6x+4+4x^2=-4x^2+4+4x^2[/tex]

[tex]\implies 6x^2+6x+4=4[/tex]

Subtract 4 from both sides:

[tex]\implies 6x^2+6x+4-4=4-4[/tex]

[tex]\implies 6x^2+6x=0[/tex]

Factor out 6x from the left side:

[tex]\implies 6x(x+1)=0[/tex]

Therefore:

[tex]\implies 6x=0 \implies x=0[/tex]

[tex]\implies x+1=0 \implies x=-1[/tex]

To find the y-coordinates of the found x-values, substitute the found values of x into one of the equations:

[tex]x=0 \implies -4(0)^2+4=4 \implies (0,4)[/tex]

[tex]x=-1\implies -4(-1)^2+4=0\implies (-1,0)[/tex]

Therefore, the solutions to the system of equations are:

(0, 4) and (-1, 0)

Answer:

Solutions:

a) x = 0, y = 4 ⇒ (0, 4)

b) x = -1, y = 0 ⇒ (-1, 0)

Step-by-step explanation:

Given system of equations:

a) y = 2x² + 6x + 4

b) y = -4x² + 4

1. Substitute the value of y in the second equation into the first equation:

⇒ -4x² + 4 = 2x² + 6x + 4

2. Solve for x:

⇒ -4x² + 4 = 2x² + 6x + 4 [subtract 4 from both sides]

⇒ -4x² + 4 - 4 = 2x² + 6x + 4 - 4

⇒ -4x² = 2x² + 6x [subtract 2x² from both sides]

⇒ -4x² - 2x² = 2x² - 2x² + 6x

⇒ -6x² = 6x [subtract 6x from both sides]

⇒ -6x² - 6x = 6x - 6x

⇒ -6x² - 6x = 0 [factor out -6x from the equation]

⇒ -6x(x + 1) = 0

Two cases:

a)

⇒ -6x = 0 [divide both sides by -6]        

⇒ -6x ÷ -6 = 0 ÷ -6

x = 0

b)

⇒ x + 1 = 0 [subtract 1 from both sides]

⇒ x + 1 - 1 = 0 - 1

x = -1

3. Find the value of y by substituting the found x values into one of the given equations:

a) x = 0:

⇒ y = -4x² + 4

⇒ y = -4(0)² + 4

⇒ y = -4(0) + 4

⇒ y = = 0 + 4

⇒ y = 4

coordinate: (0, 4)

b) x = -1:

⇒ y = -4x² + 4

⇒ y = -4(-1)² + 4

⇒ y = -4(1) + 4

⇒ y = -4 + 4

⇒ y = 0

coordinate: (-1, 0)

Learn more here:

brainly.com/question/27850666

brainly.com/question/27276129

The depreciation method that does not initially use the residual value in depreciation calculations is the.

Answers

An asset loses value over time as a result of use, damage, or obsolescence.

What is depreciation?

An asset loses value over time as a result of use, damage, or obsolescence. Depreciation is the measurement for this decline.

The depreciation method that does not initially use the residual value in depreciation calculations is the depreciating balance.

Learn more about Depreciation:

https://brainly.com/question/15085226

#SPJ1

A box has 1 red marble, 1 blue marble, and 4 green marbles of the same shape and size. dan draws a marble randomly from the box, replaces it, and then draws another marble randomly. what is the probability of drawing two green marbles in a row? explain your answer.

Answers

[tex]|\Omega|=6^2=36\\|A|=4\cdot3=12\\\\P(A)=\dfrac{12}{36}=\dfrac{1}{3}\approx33.3\%[/tex]

Answer: 1/5

Step-by-step explanation:

There are three kinds of Marbles. (M) marbles

(G) green: 4m

(R) red: 1m

(B) blue: 1m

Dan pulled a green marble out leaving

(G) green: 3m

So, the probability of picking a green marble is 1/5

sorry if I'm wrong/correct me if I'm wrong.

Casey picked 6 pounds of strawberries. Karen picked 48 ounces of blueberries. Jude picked 2 pounds of raspberries. They are going to make berry pies. The table shows the amount of berries needed for 1 pie.Select all of the true statements below.
A.

Together, Karen and Jude picked the same weight of fruit as Casey picked.


B.

Casey picked 2 times the weight of fruit that Karen picked.


C.

There are enough berries to make 6 berry pies.


D.

If they make as many pies as they can with the berries they picked, there will be 4 pounds of strawberries left over.


E.

To make 8 berry pies, they will need 1 more pound of blueberries and 2 more pounds of raspberries.

Answers

Answer:

Option B is correct

Answer:

B there is enough raspberries to make 4 pie they can 6 pies if they 2 more of raspberries

PLEASE HELP I WILL MARK BRAINLEST

Answers

#1

Error at sign shift due to -

Correct version

[tex]\\ \rm\Rrightarrow (x^3+2x^2-x)-(3x^3-3x^2+2x-4)[/tex]

[tex]\\ \rm\Rrightarrow x^3+2x^2-x-3x^3+3x^2-2x+4[/tex]

[tex]\\ \rm\Rrightarrow x^3+5x^2-3x+4[/tex]

#2

Not multiplied 3x with x

[tex]\\ \rm\Rrightarrow (x^2-3x+2)(x+1)[/tex]

[tex]\\ \rm\Rrightarrow x(x^2-3x+2)+x^2-3x+2[/tex]

[tex]\\ \rm\Rrightarrow x^3-3x^2+2x+x^2-3x+2[/tex]

[tex]\\ \rm\Rrightarrow x^3-2x^2-x+2[/tex]

hi! The work I have is solving linear equations with fractions, i only have practice exercises so they should probably be a bit simple, shown work would be appreciated <3
Just 4 questions
1) y / 3 + 6 = -3 / 2
2) -2 / 3 r - 3 = 3
3) 2 / 3 - 2x = x
4) 7x / 2 + -5 / 2 = 9/2
I’ll add a photo of the problems in a simple format

Answers

The value of the variables are y = -45/2, r = -9, x = 2/9 and x = 2

How to solve the variables?

(1) y/3 + 6 = -3/2

Multiply through by 3

y + 18 = -9/2

Subtract 18 from both sides

y = -18 - 9/2

Take LCM

y = (-18 * 2 - 9)/2

Evaluate

y = -45/2

(2) -2r/3 - 3 = 3

Multiply through by 3

-2r - 9 = 9

Add 9 to both sides

-2r = 18

Divide by 2

r = -9

(3) 2/3 - 2x = x

Multiply through by 3

2 - 6x = 3x

Add 6x to both sides

2 = 9x

Divide by 9

x = 2/9

(4) 7x/2 + -5/2 = 9/2

Multiply through by 2

7x - 5 = 9

Add 5 to both sides

7x = 14

Divide by 7

x = 2

Hence, the value of the variables are y = -45/2, r = -9, x = 2/9 and x = 2

Read more about fractions at:

https://brainly.com/question/11562149

#SPJ1

timmy makes a table to keep track of the number of visitors to his new websites as time progresses: is the data he collected best modeled by an exponential or linear function

Answers

Answer:

Show the process

A triangle is shown with its exterior angles. The interior angles of the triangle are angles 2, 3, 5. The exterior angle at angle 2 is angle 1. The exterior angle at angle 3 is angle 4. The exterior angle at angle 5 is angle 6.
Which statements are always true regarding the diagram? Select three options.

m∠5 + m∠3 = m∠4
m∠3 + m∠4 + m∠5 = 180°
m∠5 + m∠6 =180°
m∠2 + m∠3 = m∠6
m∠2 + m∠3 + m∠5 = 180°

Answers

Answer:

m∠5 + m∠6 =180°m∠2 + m∠3 = m∠6m∠2 + m∠3 + m∠5 = 180°

Step-by-step explanation:

m∠5 + m∠6 = 180° - This is true because the exterior angle and interior angle form a linear pair, and are thus supplementary.

m∠2 + m∠3 = m∠6 - This is true by the exterior angle theorem.

m∠2 + m∠3 + m∠5 = 180° - This is true because the interior angles of a triangle add to 180 degrees.

The three right options are:

m∠5 ₊ m∠6 = 180° which represents a linear pair.

∠2 ₊ ∠3 = ∠6 showing the exterior angle property of triangle.

m∠2  m∠3  m∠5 = 180° as the sum-triangle

sum-triangleproperty.

Given,

the interior angles of the triangle are 2,3,5

the exterior angle at ∠2 is ∠1.

the exterior angle at ∠3 is ∠4.

the exterior angle at ∠5 is ∠6.

Now, take into account a triangle with external angles as ∠1 , ∠4 and ∠6.

Apply the exterior angle property of a triangle.

The sum of the opposing internal angles determines the outer angle of a triangle.

For exterior ∠1 we get

∠1 = ∠5 ₊ ∠3

Similarly, for exterior ∠4 we get

∠4 = ∠5 ₊ ∠2

Then for exterior ∠6 we get

∠6 = ∠2 ₊ ∠3

∴ through the exterior angle property we get the correct option as

m∠6 = m∠2 ₊ m∠3

Now we know the sum-triangle property that sum of the angles in triangle is equal to 180°

m∠2 ₊ m∠3 ₊ m∠5 = 180°

We know the linear pair property i.e, If the angles follow the point where the two lines cross, they are considered to be linear. A linear pair's angles add up to 180° in every case.,

m∠5 ₊ m∠6 = 180°

Hence we get the statements which match the diagram are:

m∠5 ₊ m∠6 = 180°

m∠2 ₊ m∠3 ₊ m∠5 = 180°

m∠6 = m∠2 ₊ m∠3

Learn more about "triangles" here-

brainly.com/question/2644832

#SPJ10

Write the equation of the line that passes through the points (8,-1) and (2,-5) in standard form, given that the point-slope form is y+1=3/3(x-8)

Answers

The equation of the line that passes through the points (8,-1) and (2,-5) is 2x - 3y = 19.

The given coordinate are (8, -1) and (2, -5).

What is the slope of an equation?

In mathematics, the slope or gradient of a line is a number that describes both the direction and the steepness of the line.

The slope of the equation is [tex]m=\frac{y_{2}-y_{1} }{x_{2}-x_{1}}[/tex].

The standard form of an equation is Ax + By = C.

The given the point-slope form is [tex]y+1=\frac{2}{3} (x-8)[/tex].

Multiply each term by 3 of the equation.

[tex](y+1) \times3=\frac{2}{3} (x-8) \times3[/tex]

⇒3y + 3 = 2x - 16

Subtract 2x from both sides of a equation.

That is, -2x + 3y + 3 = - 16

Subtract 3 from both sides of a equation.

That is, -2x + 3y = - 19

Multiply each term by - 1 from both sides of an equation.

That is, 2x - 3y = 19

Therefore, the equation of the line that passes through the points (8,-1) and (2,-5) is 2x - 3y = 19.

To learn more about the slope visit:

https://brainly.com/question/3605446.

#SPJ1

[tex]\sqrt{6} /\sqrt{27}[/tex]

Answers

[tex]\mathrm{Apply\:the\:laws\:of\:exponents}:\quad \dfrac{\sqrt{a}}{\sqrt{b}}=\sqrt{\dfrac{a}{b }},\:\quad \:a\ge 0,\:b\ge 0[/tex]

[tex]\dfrac{\sqrt{6}}{\sqrt{27}}=\sqrt{\dfrac{6}{27}}[/tex]

[tex]=\sqrt{\dfrac{6}{27}}[/tex]

[tex]\mathrm{ Cancel \ \ \dfrac{6}{27} \ \ \ ; \ \ \dfrac{2}{9} }[/tex]

[tex]=\sqrt{\dfrac{2}{9}}[/tex]

[tex]\mathrm{Apply\:the\:laws\:of\:exponents}:\quad \sqrt{\dfrac{a}{b}}=\dfrac{\sqrt{a}}{\sqrt{b }},\:\quad \:a\ge 0,\:b\ge 0[/tex]

[tex]=\sqrt{\dfrac{2}{9}}[/tex]

[tex]\sqrt{9}=3[/tex]

[tex]=\dfrac{\sqrt{2}}{3} \ \ === > \ \ \ Answer[/tex]

[tex]\red{\boxed{\green{\boxed{\boldsymbol{\purple{Pisces04}}}}}}[/tex]

Find the nth term of this number sequence
1, 7, 13, 19, ...

Answers

Answer:

now we have to follow up with the formula n² - n¹

which is 7-1= 6

n³-n²

which is 13-7=6

so therefore the nth term=6

On a coordinate plane, a right triangle has points A (negative 5, negative 6), B (negative 5, negative 2), C (negative 2, negative 2). Examine the right triangle ABC. Which rise and run would create a similar right triangle on the same line? a rise of 6 and a run of 8 a rise of 8 and a run of 6 a rise of 6 and a run of 5 a rise of 5 and a run of 6

Answers

The rise and run that would give us a similar right triangle on the same given line is: B. a rise of 8 and a run of 6.

What is the Rise and Run of Points on the Same Line?

The rise/run of any two points on the same line are have the same slope value.

Given the coordinates:

A(-5, -6)B(-5, -2)

The rise of triangle ABC = change in y = 4 units

The run of triangle ABC = change in x = 3 units

Rise/run = 4/3

Therefore, a rise of 8 and a run of  6 would give us: 8/6 = 4/3.

This will give us a triangle similar to triangle ABC.

Therefore, the answer is: B. a rise of 8 and a run of 6.

Learn more about the rise and run of a line on:

https://brainly.com/question/14043850

#SPJ1

what is the median in this list
3 6 9 7 4 6 7 0 7
i need the median the range and the mode
ANYTHING HELPS

Answers

Median: 6

Range: 9

Mode: 7, appeared 3 times

A sequence of numbers begins with 12 and progresses geometrically. Each number is the previous number divided by 2.

Which value can be used as the common ratio in an explicit formula that represents the sequence?

One-half
2
6
12

Answers

The common ratio of the given Geometric progression is one-half.

Let a be first term and r common ratio of the Geometric progression.

The Geometric sequence is the form a,ar,ar^2, ...

Given that the sequence begins with 12, i.e a = 12

and also given that the sequence are in Geometric Progression.

By dividing the preceeding number by 2, we get the successive number

Then the Geometric sequence is 12, 12×1/2, 12×1/4, 12×1/8, 12×1/16 ...

= 12, 6, 3, 3/2, 3/4, ...

Common ratio = Second term / First term

= 6/12

=1/2

Therefore, the common ratio of the given geometric progression is one-half.

Learn more about Geometric Progression here:

https://brainly.com/question/24643676

#SPJ10

Answer:

its a

Step-by-step explanation:

Other Questions
Question 7 (True/False Worth 1 points)(05.03 LC)The correct way to calculate population density is by dividing the total population by the land area.O TrueO False Include a discussion of how development will be affected by not attaining these life tasks with respect to the developmental theorists discusses in your class notes and text (K.Warner Schae, Erikson, Levinson and Sternberg). If : f(x)=x^2, g(x)=x-1. Find (fg)(x). importance of software in computer Read the excerpt from William Shakespeares Hamlet.To be, or not to be, that is the question:Whether tis nobler in the mind to sufferThe slings and arrows of outrageous fortune,Or to take arms against a sea of troubles,And by opposing end them? To die: to sleep;No more; and by a sleep to say we endThe heart-ache and the thousand natural shocksThat flesh is heir to, tis a consummationDevoutly to be wishd. To die, to sleep;To sleep: perchance to dream: ay, there's the rub;For in that sleep of death what dreams may comeWhen we have shuffled off this mortal coil,Must give us pause: there's the respectThat makes calamity of so long life;This excerpt is an example ofblank verse.free verse.heroic couplet.common meter. A person drops off a prescription for a beneficiary who is a regular customer. The prescription is for a controlled substance with a quantity of 160. This beneficiary normally receives a quantity of 60, not 160. You review the prescription and have concerns about possible forgery. What is your next step?. PLEASE HELP MAX POINTS AND BRAINLIEST!!!(04.03 MC)A biologist is studying the growth of a particular species of algae. She writes the following equation to show the radius of the algae, f(d), in mm, after d days:f(d) = 7(1.06)dPart A: When the biologist concluded her study, the radius of the algae was approximately 13.29 mm. What is a reasonable domain to plot the growth function? (4 points)Part B: What does the y-intercept of the graph of the function f(d) represent? (2 points)Part C: What is the average rate of change of the function f(d) from d = 4 to d = 11, and what does it represent? (4 points) What is the measure of m?m = [?]Give your answer in simplest form. Which pair of products can be regarded as substitute products? can't understand tbh anyone willing give a go? describe Welhausen Elementary School and the community it served can someone please help walk me through the steps to solve number 79? im lost All of the following foods are sources of Vitamin D, except: a. salmon. b. red meats. c. fortified milk. d. sardines. e. egg yolks Blackbirding was a European policy that removed the Kanak people from the island of _______ and forcibly relocated them to ______ where they were enslaved. What is the domain of the function y=x? wich of these is a risk of speeding? For much of South and Southeast Asia, winter is a period characterized by relatively dry conditions. True False The supremacy clause gives the greatest amount of power to The scientists who have contributed to our current body of knowledge werefrom what part of the world?A. All areas of the worldB. Mostly EuropeC. Mostly the Middle EastD. Only the United StatesSEMIT Find an angle between zero and 360 that is coterminal with 900